Reversing Order of Integration: Double Integral Evaluation

Click For Summary
To evaluate the double integral by reversing the order of integration, begin by sketching the region defined by the original limits. The inner integral has x ranging from the cube root of y to 2, while the outer integral has y ranging from 0 to 8. This requires plotting the curves x = y^(1/3) and x = 2, along with the lines y = 0 and y = 8. After determining the new limits for the reversed order, the inner limits will be defined by two functions of y, and the outer limits will be based on two x values. Successfully reversing the order of integration allows for a clearer evaluation of the integral.
BrownianMan
Messages
133
Reaction score
0
Evaluate the integral by reversing the order of integration.

gif.latex?\int_{0}^{8}\int_{\sqrt[3]{y}}^{{2}}7e^{x^4}dxdy.gif


I'm not exactly sure how to approach this problem. Any help would be appreciated!
 

Attachments

  • gif.latex?\int_{0}^{8}\int_{\sqrt[3]{y}}^{{2}}7e^{x^4}dxdy.gif
    gif.latex?\int_{0}^{8}\int_{\sqrt[3]{y}}^{{2}}7e^{x^4}dxdy.gif
    872 bytes · Views: 351
Physics news on Phys.org
BrownianMan said:
Evaluate the integral by reversing the order of integration.

gif.latex?\int_{0}^{8}\int_{\sqrt[3]{y}}^{{2}}7e^{x^4}dxdy.gif


I'm not exactly sure how to approach this problem. Any help would be appreciated!
The first step in this type of problem is to sketch a graph of the region over which integration is taking place. For the inner integral, x ranges from cuberoot(y) to 2. For the outer integral, y ranges from 0 to 8. Sketch the graphs of x = y1/3 and x = 2, and then sketch the graphs of y = 0 and y = 8. For the iterated integral with the opposite order, the inner integration limits will involve two functions of y, and the outer integration limits will involve two x values.
 
Ok, so would the answer be

gif.latex?\frac{7}{4}\left%20(%20e^{16}-1%20\right%20).gif
 
Question: A clock's minute hand has length 4 and its hour hand has length 3. What is the distance between the tips at the moment when it is increasing most rapidly?(Putnam Exam Question) Answer: Making assumption that both the hands moves at constant angular velocities, the answer is ## \sqrt{7} .## But don't you think this assumption is somewhat doubtful and wrong?

Similar threads

Replies
10
Views
2K
  • · Replies 17 ·
Replies
17
Views
4K
  • · Replies 9 ·
Replies
9
Views
2K
Replies
2
Views
1K
  • · Replies 7 ·
Replies
7
Views
2K
Replies
18
Views
2K
  • · Replies 10 ·
Replies
10
Views
2K
  • · Replies 3 ·
Replies
3
Views
2K
Replies
4
Views
1K
  • · Replies 4 ·
Replies
4
Views
2K